The Muller family are on holiday in New Zealand. a. They change some euros (€) and receive $1962 (New Zealand dollars). The exchange rate is €1 = $1.635. Calculate the number of euros they change. [3] b. The family spend 15% of their New Zealand dollars on a tour. Calculate the number of dollars they have left. [4]

Answers

Answer 1

Answer:

a. €1200;$1667.70

Step-by-step explanation:

a. Number of euros

[tex]\text{euros} = \$1962 \times \dfrac{\text{1 euro}}{\text{\$1.635}} = \textbf{1200 euros}[/tex]

b. Dollars remaining

Dollars on hand                        = $1962.00

Less 15 % spent = 0.15 × 1962 =   -294.30

Balance remaining                   =  $1667.70


Related Questions

HELP ME PLEASE The student's in Roberto's school are painting a mural that will be 8 feet by 15 feet. First they make a scale drawing of the mural with a scale of 2 feet:5 feet. What are the length and width of the scale drawing in feet

Answers

Answer:

Length = 3.2feets ; width = 6feets

Step-by-step explanation:

Given the following :

Actual dimension of Mural:

8feets by 15feets

Scale drawing ratio of Mural :

2feets : 5feets

This means :

2 Feets on paper equals 5 Feets of actual drawing:

Therefore, with actual dimension of Length = 8feets ;

Length of scale drawing = (2/5) * 8 = 3.2feets

Actual dimension of width 15feets ;

Width of scale drawing = (2/5) * 15 = 6 Feets

The expansion of (1 + 4x)5 is​

Answers

Answer:

5+20x

Step-by-step explanation:

multiply 5 to 1 and 4x respectively

Each lap around a park is 1 1⁄5 miles. Kellyn plans to jog at least 7 1⁄2 miles at the park without doing partial laps. How many laps must Kellyn jog to meet her goal?

Answers

Answer:

25/4 laps or (6.25 laps)

Step-by-step explanation:

1 lap = 1 1/5 miles

kellyn plans to jog 7 1/2 miles

                                                      1  lap

number of laps = 7 1/2 miles x  --------------   = 25/4 laps or (6.25 laps)

                                                    1 1/5 miles

A principal of $2600is invested at 6.75% interest, compounded annually. How much will the investment be worth after 14 years

Answers

Answer:

$6488.19

Step-by-step explanation:

To solve this problem we use the compounded interest formula:

[tex]amount = principal(1 + (r \n))^({n}{t})[/tex]

a = $2600(1+(0.0675/1))¹*¹⁴

a = $6488.19

A box has a base of 12 inches by 12 inches and a height of 30 inches. What is the length of the interior diagonal of the box? Round to the nearest hundredth.

Answers

This is a problem using the Pythagorean theorem.

The square of the length required is 122 + 122 + 302 = 1188

 

The length is the square root of this number; I will leave it to you to extract the square root.

Answer:

34.47

Step-by-step explanation:

HE SODA MACHINE
The soda machine at your school offers several types of soda. There are two buttons for your favorite drink, Blast,
while the other drinks (Slurp, Lemon Twister, and Diet Slurp) each have one button.
Describe the input and output of this soda machine.
While buying a soda, Ms. Whitney pushed the button for Lemon Twister and got a can of Lemon Twister.
Later she went back to the same machine, again pushing the Lemon Twister button, but this time she got a can
of Blast. Is the machine functioning consistently? Why or why not?
BLAST
BLAST
BLUSO
ooo
Students, write your resnonsel

Answers

Answer:

yes and no

Step-by-step explanation:

if a machine is work a lot lot LOT, then it might be jamed

or the teacher might have accidentally pressed blast and you might have not seen the buttons correctly

( also i am not a student of yours )

The focus of a parabola is (3,-7) and the directrix is y = -4.
What is an equation of the parabola?

Answers

Answer:

  (a)  (x -3)^2 = -6(y +5.5)

Step-by-step explanation:

The equation of a parabola can be written as ...

  (x -h)^2 = 4p(y -k)

where (h, k) is the vertex, and p is the distance from the focus to the vertex.

The vertex is half-way between the focus and directrix, so is ...

  (h, k) = (1/2)((3, -7) +(3, -4)) = (3, -5.5)

The focus is at y=-7, and the vertex is at y=-5.5, so the distance between them is ...

  -7 -(-5.5) = -1.5

Then the equation for the parabola is ...

  (x -3)^2 = 4(-1.5)(y -(-5.5))

  (x -3)^2 = -6(y +5.5) . . . . matches the first choice

A file that is 276 megabytes is being dowloaded . If the downloaded is 16.7% complete how many megabytes have been dowloaded? Round ur answer to the nearest tenth ( can ya please hurry and answer thank you)

Answers

46.1 megabytes
276 x .167 = 46.092

Answer: 46.1 megabytes

Step-by-step explanation:

What is the slope of the line? x + 3 y = 10 x+3y=10x, plus, 3, y, equals, 10 Choose 1 answer: Choose 1 answer: (Choice A) A 1 3 3 1 ​ start fraction, 1, divided by, 3, end fraction (Choice B) B 1 10 10 1 ​ start fraction, 1, divided by, 10, end fraction (Choice C) C − 1 10 − 10 1 ​ minus, start fraction, 1, divided by, 10, end fraction (Choice D) D − 1 3 − 3 1 ​

Answers

Answer:

-1/3

Step-by-step explanation:

The standard from of equation of a line written in slope-intercept format is expressed as y = mx+c where c is the slope of the line and c is the y-intercept.

Given the equation of the line x+3y = 10, to get the slope of the line, we need write he equation in standard from first by making y the subject of the formula as shown;

[tex]x+3y = 10\\\\subtract\ x \ from \ both \ sides\\\\x+3y-x = 10 -x\\\\3y = -x+10\\\\Divide \ through\ by \ 3\\\\\frac{3y}{3} = -\frac{x}{3} +\frac{10}{3} \\\\[/tex]

[tex]y = -\frac{1}{3}(x) +\frac{10}{3} \\[/tex]

Comparing the resulting equation with y = mx+c, the slope 'm' of the equation is -1/3

The slope of the line is -1/3.

the correct option is D.

To find the slope of the line given by the equation x + 3y = 10, we need to rewrite it in slope-intercept form, which is y = mx + b, where m represents the slope.

Let's rearrange the equation to solve for y:

x + 3y = 10

3y = -x + 10

y = (-1/3)x + 10/3

Comparing this equation to the slope-intercept form, we can see that the coefficient of x (-1/3) represents the slope.

Therefore, the slope of the line is -1/3.

Learn more about Slope here:

https://brainly.com/question/17158875

#SPJ6

Between what two consecutive integers on the number line is the graph of the sum sqrt(30) + sqrt(50)?

Answers

Answer:

sqrt(30)+sqrt(50) = 12.5482933869171364     which is between   m and n    12 and 13

Question 9(Multiple Choice Worth 4 points)
(02.05 LC)
Which of the following statements best describes the effect of replacing the
graph of y = f(x) with the graph of y = f(x - 9)?
The graph of y = f(x) will shift up 9 units.
The graph of y = f(x) will shift down 9 units.
The graph of y = f(x) will shift left 9 units.
The graph of y = f(x) will shift right 9 units.

Answers

Answer:  D) right 9 units

Step-by-step explanation:

The Vertex form of a quadratic equation is: y = a(x - h)² + k     where

a is the vertical stretch or shrink(x, h) is the vertex

        → h is the horizontal shift (+ is right, - is left)

        → k is the vertical shift (+ is up, - is down)

y = f(x - 9)

            ↓

           h=9

Since h is positive, the graph moves 9 units to the RIGHT

Answer:

The graph of y = f(x) will shift right 9 units.

Step-by-step explanation:

If the -9 is inside the parenthesis, the graph g(x) shifts 9 points to the right.

Evaluate 2^2⋅4^3=

Your answer


Answers

Answer:

256

Step-by-step explanation:

First, handle the exponent:

2²=4 (2*2=4)     and     4³=64 (4*4=16*4=64)

Now multiply those two outcomes:

4*64=256

This equation is also known as 4⁴

the cost of cementing a wall 8 feet wide and 24 feet long at 14.40 a square yard is

will mark brainlist

Answers

Answer:

$307.20

Step-by-step explanation:

8×24=192

Convert 192 square feet to square yards, which is 21.3333

Then multiple: 21.333×14.40=307.1999

Then round to the nearest hundredths

The final answer is $307.20



While sailing, Jacob is 150 feet from a lighthouse. The angle of elevation from his feet on the boat (at sea level) to the top of the lighthouse is 48°

What can you conclude about the height of the lighthouse, with respect to Jacob's distance from the lighthouse? Explain your answer.

Answers

Answer:

The height of the lighthouse is approximately 166.6 feet.

Step-by-step explanation:

Let the height of the lighthouse be represented by s, then;

Tan 48° = (opposite) ÷ (adjacent)

Tan 48° = s ÷ 150

⇒ s = 150 × Tan 48°

      = 150 × 1.1106

     = 166.59

s ≅ 166.6 feet

Therefore, the height of the lighthouse is approximately 166.6 feet.

Answer:

The height of the lighthouse is approximately 42.

Step-by-step explanation:

This situation forms a right triangle so the angles would be 90 and 48

90+48+x=180

138+x=180

x=180-138

x=42

an inch worm is how long in general

Answers

Answer:

A inch

Step-by-step explanation:

0.0000458 as scientific notation

Answers

Answer:

4.58 * 10 ^ -5

Step-by-step explanation:

0.0000458

Move the decimal 5 places to the right to get a number between 1 and less than 10

00004.58

That will give us an exponent of -5 ( the negative is because we moved it to the right)

4.58 * 10 ^ -5

━━━━━━━☆☆━━━━━━━

▹ Answer

4.58 * 10⁻⁵

▹ Step-by-Step Explanation

The decimal point is moved 5 times to the left, meaning the scientific notation will be negative. Therefore, the answer is 4.58 * 10⁻⁵.

Hope this helps!

CloutAnswers ❁

━━━━━━━☆☆━━━━━━━

I got this wrong please help me get the right answer in the first little box marked in red. thx

Answers

Answer:

The inverse is 1/2x +2

Step-by-step explanation:

f(x) = 2x-4

y= 2x-4

To find the inverse

Exchange x and y

x = 2y-4

Solve for y

Add 4 to each side

x+4 = 2y

Divide by 2

x/2 + 4/2 = 2y/2

1/2x +2 = y

The inverse is 1/2x +2

Four witches are making a brew.Patricia puts in x frogs legs.sheila puts in five more than patricia.Anne puts double the amount that Sheila adds,and Winifred adds three times the number Sheila adds.
write down an algebraic expression for:
Sheila. Anne. Winifred

Answers

Answer:

see explanation

Step-by-step explanation:

Given Patricia puts in x frogs legs, then

Sheila puts in x + 5 ( 5 more than Patricia )

Anne puts in 2(x + 5) ( double the amount of Anne )

Winifred puts in 3(x + 5) ( three times the number of Sheila )

Pls answer this urgently

Answers

Answer:

The pic there has a step by step explanation

●✴︎✴︎✴︎✴︎✴︎✴︎✴︎✴︎❀✴︎✴︎✴︎✴︎✴︎✴︎✴︎✴︎✴︎●

        Hi my lil bunny!

❧⎯⎯⎯⎯⎯⎯⎯⎯⎯⎯⎯⎯⎯⎯⎯⎯⎯⎯⎯⎯⎯⎯⎯⎯⎯⎯⎯⎯⎯⎯⎯⎯⎯⎯⎯⎯⎯⎯☙

[tex]\frac{(3^-2)^2 * (5^2)^-3 * ( t ^-3)^2 }{( 3^-2) ^5 ( 5^3) ^-2 ( t ^-4)^3}[/tex]

[tex]= \frac{\frac{1}{1265625^t12} }{1}[/tex]

[tex]= \frac{\frac{1}{992640625^t6} }{1}[/tex]

[tex]= \frac{992640625^t6}{1265625}[/tex]

[tex]= \frac{922640625tttttt}{1265625}[/tex]

[tex]= 729t^6[/tex]

Answer : [tex]729t^6[/tex]

❧⎯⎯⎯⎯⎯⎯⎯⎯⎯⎯⎯⎯⎯⎯⎯⎯⎯⎯⎯⎯⎯⎯⎯⎯⎯⎯⎯⎯⎯⎯⎯⎯⎯⎯⎯⎯⎯⎯☙

●✴︎✴︎✴︎✴︎✴︎✴︎✴︎✴︎❀✴︎✴︎✴︎✴︎✴︎✴︎✴︎✴︎✴︎●

If this helped you, could you maybe give brainliest..?

❀*May*❀

What is the value of x in the equation 3x-4y=65, when y =4 will give brainliest

Answers

Hello!

Answer:

[tex]\huge\boxed{x = 27}[/tex]

Given:

3x - 4y = 65 where y = 4;

Substitute in 4 for "y":

3x - 4(4) = 65

Simplify:

3x - 16 = 65

Add 16 to both sides:

3x - 16 + 16 = 65 + 16

3x = 81

Divide both sides by 3:

3x/3 = 81/3

x = 27.

Hope this helped you! :)

Answer:

x=27

Step-by-step explanation:

3x-4y=65

Let y=4

3x - 4(4) = 54

3x -16 = 65

Add 16 to each side

3x -16+16 = 65+16

3x = 81

Divide each side by 3

3x/3 =81/3

x =27

find the sum of partial fraction for x^4/(x-1)(x²-4)​

Answers

Answer:

see attachment

Step-by-step explanation:

Find the solutions to $x^2 + 64 = 0.$[tex]Find the solutions to $x^2 + 64 = 0.$[/tex]

Answers

Answer:

x=8i,\:x=-8i

Step-by-step explanation:

x^2+64-64=0-64

x^2=-64

x=\sqrt{-64},\:x=-\sqrt{-64}

x=8i,\:x=-8i

Someone please help! Thank you

Answers

I think the answer would be D, but I’m not sure.

Answer:

Hey there!

We can write a equations here:

3x+y=180

Also, since all of the angles have 3x on the outside, then y must be constant.

3y=180

y=60

Thus, for x, we have 3x+60=180, 3x=120, x=40.

2x+6y

2(40)+6(60)

80+360

440.

Let me know if this helps :)

If area of a rhombus is 336 cm and one of its diagonal is 14 cm, find its perimeter.

Answers

Answer:

The perimeter of the Rhombus is 100 cm

Step-by-step explanation:

First of all, we will need to find the length of the other diagonal.

let’s call the diagonals p and q

Mathematically, the area of the Rhombus is;

pq/2 = Area of Rhombus.

Let’s call the missing diagonal p

So;

(p * 14)/2 = 336

14p = 672

p = 672/14

p = 48 cm

Now, we can find the perimeter of the Rhombus using these diagonals.

Mathematically;

P = 2 √(p^2 + q^2)

Substituting these values, we have;

P = 2 √(14)^2 + (48^2)

P = 2 √(2500)

P = 2 * 50

P = 100 cm

The perimeter of the rhombus is the sum of its side lengths

The perimeter of the Rhombus is 100 cm

The length of one of its diagonal is given as:

[tex]p= 14[/tex]

And the area is given as:

[tex]A = 336[/tex]

Assume the other diagonal is q.

The area of the rhombus is represented as:

[tex]A = \frac{pq}2[/tex]

So, we have:

[tex]336 = \frac{14q}2[/tex]

This gives

[tex]336 = 7q[/tex]

Divide both sides by 7

[tex]48 = q[/tex]

Rewrite as:

[tex]q = 48[/tex]

The perimeter (P) of the rhombus is calculated as:

[tex]P =2\sqrt{p^2 + q^2[/tex]

So, we have:

[tex]P =2\sqrt{48^2 + 14^2[/tex]

Evaluate the squares

[tex]P =2\sqrt{2500[/tex]

Take positive root of 2500

[tex]P =2 \times 50[/tex]

[tex]P =100[/tex]

Hence, the perimeter of the Rhombus is 100 cm

Read more about areas and perimeters at:

https://brainly.com/question/14137384

Please answer this question now

Answers

Answer:

54 degrees

Step-by-step explanation:

Measure of arc DCB is 125*2 = 250.

So, measure of arc BAD is 360-250 = 110.

So, measure of arc AD is 110 - 56 = 54 degrees

Write an expression to represent the product of 6 and the square of a number plus 15.

Answers

Answer:

6x^2 + 15

Step-by-step explanation:

We don't know what the square of a number is, so we represent it as x^2

The first part tells us to find the product of 6 and the square of a number, so

6x^2

The second part tells us to add 15 so,

6x^2 + 15

The expression that should be represented is [tex]6x^2 + 15[/tex]

Given that

The product of 6.The square of number + 15.

Based on the above information,

Let us assume the number should be x

So, from the above information, the expression should be [tex]6x^2 + 15[/tex]

Therefore we can conclude that The expression that should be represented is [tex]6x^2 + 15[/tex]

Learn more about the equation here:brainly.com/question/21105092?

help me with guys plzzzzz

Answers

Answer:

C

Step-by-step explanation:

So the lion weighs more than the ant right?

And we wish to figure out how much greater the lion's weight is compared to the ant's weight.

So only thing we have to do is divide.

[tex]\frac{1 * 10^5}{2*10^-3} = \frac{10^5}{2*10^-3} = \frac{10^8}{2} = 50000000 = 5 * 10^7[/tex]

Answer:

5×10^7

Step-by-step explanation:

2×10^-3    1×10^5

(1×10^5)/(2×10^-3)=

5×10^7

to scheck :(2*10^-3)(5*10^7)=(1*10^5)

solve for x 13x + 7 = 5x - 20

Answers

Answer:

13x + 7 = 5x - 20

Step-by-step explanation:

SOLVE for X

13x + 7 = 5x - 20

-------------------------

-5x                   -7  

------------------------

8x=-27

x= -3.375 or -27/8

Answer:

x = -27/8

Step-by-step explanation:

combine the x terms.  To do this, subtract 5x from both sides.  We get:

8x + 7 = -20.  

Next, subtract 7 from both sides, obtaining:

8x = -27, or x = -27/8

Suppose you roll a fair six-sided die 25 times. What is the probability that you roll 5 or more 6’s on that die?

A. 0.3883

B. 0.5937

C. 0.5

D. 0.4063

Answers

Answer:

D. P(5+ 6's) = 0.4063

Step-by-step explanation:

Binomial distribution.

For the distribution to be applicable, the experiment must

1. Have a know and constant number of trials

2. Probability of success of each trial remains constant (and known if available)

3. Each trial is a Bernoulli trial, i.e. with only two outcomes, success or failure.

4. Independence between trials.

Let  

n = number of trials  = 25

p = probability of success of each trial  = 1 / 6

x = number of successes  (0 ≤ x ≤ n)  = 5

C(n,x) = number of combinations of picking x identical objects out of n

Applying binomial distribution

P(x,n) = probability of x successes in an experiment of n trials.

= C(n,x) * p^x * (1-p)^(n-x)

For n = 25 trials with probability of success (roll a 6) = 1/6

and x = 5,6,7,8,...25

It is easier to calculate the complement by

P(5+ 6's) = 1 - P(<5 6's)

= 1 - ( P(0,25) + P(1,25) + P(2,25) + P(3,25) + P(4,25) )

1- (

    P(0,25) = C(25,0) * (1/6)^0 * (5/6)^25 = 0.0104825960103961

    P(1,25) = C(25,1) * (1/6)^1 * (5/6)^24 = 0.05241298005198051

     P(2,25) = C(25,2) * (1/6)^2 * (5/6)^23 = 0.1257911521247532

    P(3,25) = C(25,3) * (1/6)^3 * (5/6)^22 = 0.1928797665912883

    P(4,25) = C(25,4) * (1/6)^4 * (5/6)^21 = 0.2121677432504171

)

= 1 - 0.59373

= 0.40626

= 0.4063 (to 4th decimal place)

Nan left her house at 1 p.m. driving at a constant speed of 40 miles perhour. If she continues at that same speed for the entire trip, she will reach
her destination at 4:45 p.m. If Nan were to drive 10 miles per hour faster,
how much quicker would she arrive?

Answers

Answer:

45 minutes faster

Step-by-step explanation:

Distance:

40 x 3.75 = 150

If travel 50 miles/h:

150/50 = 3

3.75 - 3 = 0.75 of hour

45 minutes

Hope that helped!!! k

Other Questions
every rational number is aa. whole number b. natural number c. integer d. real number PLZ HELPPPPPP. 25 POINTS. A store sells books for $12 each. In the proportional relationship between x, the number of books purchased, and y, the cost per books in dollars" to "y, the total cost of the books in dollars, the constant of proportionality is 12. Which equation shows the relationship between x and y? A. y=12/x B. y=12x C. y=12+x D. y=12x please help !! Solve 2.5x 25 A student did an experiment with two identical fish tanks, Tank 1 and Tank 2. About 20 ml water purifier was added to Tank 1 and 10 mlwater punfier was added to tank 2. Tank 1 was heated to 88 F and Tank 2 was left at room temperature. The water purity of each tank wastested every day in particles per liter. The purity was recorded over a period of 5 weeks in the table shown below.Week Water Purity of Tank 1 Water Purity of Tank 212.612.613.713.914.314.415.715.817.617.6Based on the table, which of these conclusions is correct?aFish need 10 mL of water purifier to survive.bMore purifier is needed to keep heated water clean.Water purifier can help a fish survive longer.dWater purifier keeps fish tanks clearer for longer.pleasee helppp !! . A particular parcel of real estate (land) is sold for $20,000,000 and was originally purchased for $10,000,000. On a taxable sale, explain a circumstance (type of investor, intent, entity, etc.) that would pay the following U.S. federal income tax results on the $10,000,000 gain (exclude the 3.8% net investment income tax and any state taxes in the calculation): 2/3a - 1/6 =1/3 please help me Lauren is a college sophomore majoring in business. This semester Lauren is taking courses in accounting, economics, management information systems, public speaking, and statistics. The sizes of these classes are, respectively, 375, 35, 45, 25, and 60.Required:Find the mean and the median of the class sizes. What is a better measure of Lauren's "typical class size"the mean or the median? Who painted the image below? A painting titled Hunters in the Snow (Winter). Hunters walk through a snow-covered town while people skate on a lake covered by ice. Two parallel metal plates, each of area A, are separatedby a distance 3d. Both are connected to ground and each plate carries no charge. A third plate carrying charge Qis inserted between the two plates, located a distance dfrom the upper plate. As a result, negative charge is induced on each of the two original plates. a) In terms of Q, find the amount of charge on the upper plate, Q1, and the lower plate, Q2. (Hint: it must be true that Q Browse this website documenting the Haymarket Square Riot of 1886. Once you have finished reading some of the materials, write a short reflective essay (2-3 paragraphs) about whether it was fair for the eight men to be put on trial for the riot. Be sure to include your opinion, using specific examples from the lesson and from the web resource. Empirical evidence from 1960 to 2010 shows that convergence in economic growth is occurring in which of the following cases?a. All low-income countries are catching up to all high-income countries. b. Low-income industrial countries are catching up to high-income developing countries. c. Low-income developing countries are catching up to high-income industrial countries. d. Low-income industrial countries are catching up to high-income industrial countries. Johnstones Realty is a new discount, menu-based brokerage firm. Its approach usually results in a lower listing commission rate than other firms offer. Johnstones also offers a lower cooperating brokerage split than most other firms. Business is booming for Johnstone's, and the remaining firms in town are concerned. These brokers agree to not show Johnstones listings to their buyer clients. What is this an example of? A firm has a debt-to-equity of 0.69 and a market-to-book ratio of 3.0. What is the ratio of the book value of debt to the market value of equity? Which of the following images shows a scale copy of the trapezoid using a scale factor of 1/2PLEASE HELP An investigator claims, with 95 percent confidence, that the interval between 10 and 16 miles includes the mean commute distance for all California commuters. To have 95 percent confidence signifies that What is trade??????????? Scouts of ABC school made to run around a regular hexagonal ground fig 9, of perimeter 270 m .If they started running from point X and covered two fifth (2/5th) of the total distance.Which side of the ground will they reach? Food Shoppe Galore had the following information: Total market value of a companys stock: $650 million Total market value of the companys debt: $150 million What is the weighted average of the companys debt? Let REPEAT TM = { | M is a TM, and for all s L(M), s = uv where u = v }. Show that REPEATTM is undecidable. Do not use Rices Theorem. Twice the difference of a number and 9 is 3. Use the variable b for the unknown number.